chike_eze
Thanks Received: 94
Atticus Finch
Atticus Finch
 
Posts: 279
Joined: January 22nd, 2011
 
 
trophy
Most Thanked
 

Q16 - Among multiparty democracies, those with

by chike_eze Wed May 11, 2011 6:08 am

I'll take a shot at explaining this one, which I managed to mess up on the prep test because I lost track of the conclusion. But upon review, it seems much clearer.

Conclusion: Among multiparty... those with fewest parties will
have the most-productive legislatures

Why? Fewer parties: more # issues
more # issues: must prioritize issues
prioritize issues: Promotes tendency to compromise

Assumption: tendency to compromise <linked to> productive legislatures

Answer Choices:
A) More political parties... (Out of scope)
B) Fewer parties... compromise... (I think Premise booster)

C) Tendency to compromise > more productive legislative process
(Correct!)

D) Non democracies (Out of Scope)
E) Legislators not agree... (Irrelevant)
Last edited by chike_eze on Tue Aug 09, 2011 7:43 pm, edited 2 times in total.
User avatar
 
bbirdwell
Thanks Received: 864
Atticus Finch
Atticus Finch
 
Posts: 803
Joined: April 16th, 2009
 
 
 

Re: Q16 - Among multiparty democracies, those with

by bbirdwell Sat May 14, 2011 3:03 pm

Nice! (and I like the color-coding). Contributing explanations is a great way to hone your skills. Keep it up!
I host free online workshop/Q&A sessions called Zen and the Art of LSAT. You can find upcoming dates here: http://www.manhattanlsat.com/zen-and-the-art.cfm
User avatar
 
LSAT-Chang
Thanks Received: 38
Atticus Finch
Atticus Finch
 
Posts: 479
Joined: June 03rd, 2011
 
 
trophy
Most Thankful
trophy
First Responder
 

Re: Q16 - Among Multiparty

by LSAT-Chang Tue Aug 09, 2011 6:07 pm

I was able to end up with (C) on my 2nd try, but is this diagramming correct?

More issues
+
Needs to prioritize
+
Needs to compromise

THEREFORE (---->)

Fewest parties most productive

So the gap that I spotted was the author is assuming "more issues -> most productive" "prioritize on issues -> most productive" and "needs to compromise -> most productive", so I thought the author was assuming 3 things; and so when I saw (C), it was one of them, so I circled it. But now that I look at the argument, it seems to be that those 3 variables are linked to one another such that we have:

fewer # parties -> more issues -> prioritize issues -> tendency to compromise

So does the author not have to assume for example, "prioritize issues -> most productive"? I just wanted to make sure that I wasn't doing something silly here -- and run into a question which has all 3 of those answers that I thought were possible, and turns out to be the case that it was just one of them that was necessary!
 
timmydoeslsat
Thanks Received: 887
Atticus Finch
Atticus Finch
 
Posts: 1136
Joined: June 20th, 2011
 
 
trophy
Most Thanked
trophy
First Responder
 

Re: Q16 - Among Multiparty

by timmydoeslsat Tue Aug 09, 2011 6:27 pm

changsoyeon Wrote:I was able to end up with (C) on my 2nd try, but is this diagramming correct?

More issues
+
Needs to prioritize
+
Needs to compromise

THEREFORE (---->)

Fewest parties most productive

So the gap that I spotted was the author is assuming "more issues -> most productive" "prioritize on issues -> most productive" and "needs to compromise -> most productive", so I thought the author was assuming 3 things; and so when I saw (C), it was one of them, so I circled it. But now that I look at the argument, it seems to be that those 3 variables are linked to one another such that we have:

fewer # parties -> more issues -> prioritize issues -> tendency to compromise

So does the author not have to assume for example, "prioritize issues -> most productive"? I just wanted to make sure that I wasn't doing something silly here -- and run into a question which has all 3 of those answers that I thought were possible, and turns out to be the case that it was just one of them that was necessary!


Very good question. This is pretty advanced stuff!

You are right on with your diagramming. When I just did this problem, I did not even diagram. You will learn what questions will be best served with diagramming. You will find that on 99% of necessary assumption questions, diagramming will not be necessary.

As for your question concerning the three necessary conditions that are present in the logic chain:

fewer # parties -> more issues -> prioritize issues -> tendency to compromise

The argument uses this chain as evidence to prove the first sentence, the main conclusion of the argument.

Main conclusion: Among multiparty democracies, those with the fewest parties will have the most-productive legislatures.

And as you have pointed out, the author is assuming something for that chain to connect with the idea of having the most-productive legislatures.

Let us break this down to variables.

A ---> B ---> C ---> D

Statement X

I would like to have an instructor confirm this, but I believe that it is not necessary for statement X to be linked with either B, C.

As there can be other avenues for variable D to be reached other than B or C, or even A for that matter.
 
chike_eze
Thanks Received: 94
Atticus Finch
Atticus Finch
 
Posts: 279
Joined: January 22nd, 2011
 
 
trophy
Most Thanked
 

Re: Q16 - Among Multiparty

by chike_eze Tue Aug 09, 2011 7:35 pm

changsoyeon Wrote:fewer # parties -> more issues -> prioritize issues -> tendency to compromise

So does the author not have to assume for example, "prioritize issues -> most productive"? I just wanted to make sure that I wasn't doing something silly here -- and run into a question which has all 3 of those answers that I thought were possible, and turns out to be the case that it was just one of them that was necessary!

I may be wrong, but the way this prompt reads, the statements after the conclusion are facts (evidence). Therefore, for the most part, we are to accept them as true.
- fewer # parties, more issues
- more issues --> prioritize
- prioritize --> tendency to compromise

I think the issue is that even if we accept these statements to be true, nothing in the prompt connects "prioritizing" or "tendency to compromise" with "most-productive" legislatures.

Conclusion: fewest parties will have the most productive legislatures!
> Really, How do you know that?

Premise: Because fewer # parties have a tendency to compromise
> Okay, but so what? You haven't said anything about why fewer parties are most-productive...

The Gap: fewer # parties --> [ ? ] --> most-productive.

(C) fewer # parties --> tendency to compromise --> most-productive
User avatar
 
LSAT-Chang
Thanks Received: 38
Atticus Finch
Atticus Finch
 
Posts: 479
Joined: June 03rd, 2011
 
 
trophy
Most Thankful
trophy
First Responder
 

Re: Q16 - Among Multiparty

by LSAT-Chang Wed Aug 10, 2011 2:16 pm

How about the other variables? I understand that fewer parties --> tendency to compromise --> most productive (as it can be followed by the conditional chain), but like you said:

- fewer # parties, more issues
- more issues --> prioritize
- prioritize --> tendency to compromise

So my question is.. do we not have to assume:

fewer # parties --> more issues --> most productive
fewer # parties --> prioritize --> most productive

The question that I asked in the previous post was about whether we had to assume those as well since I didn't want to run into a problem where it gave me all 3 of them (including the actual correct answer here: fewer # parties --> tendency to compromise --> most productive) and have it be that this is the only one that is "necessary". Do I make sense :?: Just want to know the idea behind this for future reference..
 
chike_eze
Thanks Received: 94
Atticus Finch
Atticus Finch
 
Posts: 279
Joined: January 22nd, 2011
 
 
trophy
Most Thanked
 

Re: Q16 - Among Multiparty

by chike_eze Wed Aug 10, 2011 2:56 pm

changsoyeon Wrote:So my question is.. do we not have to assume:

fewer # parties --> more issues --> most productive
fewer # parties --> prioritize --> most productive

The question that I asked in the previous post was about whether we had to assume those as well since I didn't want to run into a problem where it gave me all 3 of them (including the actual correct answer here: fewer # parties --> tendency to compromise --> most productive) and have it be that this is the only one that is "necessary". Do I make sense :?: Just want to know the idea behind this for future reference..

I think I get your point now. Correct me if I'm wrong.

The conclusion states:
fewer # parties --> most productive

And our premises are:
fewer # parties --> more issues
fewer # parties --> prioritize
fewer # parties --> tend to compromise

Therefore, Possible Answers:
1. more issues --> most productive
2. prioritize --> most productive
3. tend to compromise --> most productive

Assuming we diagrammed the conditions correctly, I'd have to agree with you that either one would work. Actually, when I did this PT question, I expected to see # 2 or 3. I didn't expect # 1 because I've noticed more inferences similar to 2 and 3 on my practice sets.

Still struggling a little bit with inferences because I tend to over-think them. Thanks for pointing this stuff out... wheww!!
User avatar
 
LSAT-Chang
Thanks Received: 38
Atticus Finch
Atticus Finch
 
Posts: 479
Joined: June 03rd, 2011
 
This post thanked 1 time.
 
trophy
Most Thankful
trophy
First Responder
 

Re: Q16 - Among Multiparty

by LSAT-Chang Wed Aug 10, 2011 4:19 pm

chike_eze Wrote:
Assuming we diagrammed the conditions correctly, I'd have to agree with you that either one would work. Actually, when I did this PT question, I expected to see # 2 or 3. I didn't expect # 1 because I've noticed more inferences similar to 2 and 3 on my practice sets.

Still struggling a little bit with inferences because I tend to over-think them. Thanks for pointing this stuff out... wheww!!


Yay! Someone agrees with me. I thought so too since it would be like having:

A -> B -> C -> D
-------------------
A -> E

So we would have to connect E to the end of the chain in D.

So we would get:

A -> B -> C -> D -> E

So any of these would be the correct answer:

B -> E
C -> E
D -> E (and this is what we got!)
 
chike_eze
Thanks Received: 94
Atticus Finch
Atticus Finch
 
Posts: 279
Joined: January 22nd, 2011
 
 
trophy
Most Thanked
 

Re: Q16 - Among Multiparty

by chike_eze Wed Aug 10, 2011 4:24 pm

changsoyeon Wrote:A -> B -> C -> D
-------------------
A -> E

So we would have to connect E to the end of the chain in D.

So we would get:

A -> B -> C -> D -> E

So any of these would be the correct answer:

B -> E
C -> E
D -> E (and this is what we got!)

I like this inference chain. It works!
User avatar
 
noah
Thanks Received: 1192
Atticus Finch
Atticus Finch
 
Posts: 1541
Joined: February 11th, 2009
 
This post thanked 1 time.
 
 

Re: Q16 - Among Multiparty

by noah Thu Aug 11, 2011 11:30 am

Hey Y'all, great discussion. To add my 2 cents, you're right that in this problem:

A -> B --> C --> D
Conclusion A --> E

Any assumption connecting A, B or C to E would technically work. But, remember that the LSAT is presenting an argument, so the idea is to link the premise to the conclusion, and so it's much more representative of what the LSAT is testing to require you use all your supporting premises in this sort of problem. Also, be aware that there could be a slight variation in the premise chain: (A --> B) --> C --> D, concluding A --> D and the connection is totally different.

(This would sound something like this: When our Apple harvest leads to an abundance of Bruised apples, the Cat gets fed those apples, which exacerbates its Diabetes, which surprisingly leads it to have more Energy. So, the cat will have more Energy during our Apple harvest. Here, the assumption is that the harvest inevitably leads to an abundance of bruised apples, i.e that A always leads to B.)

However, the important thing to know, since we're talking about all of this in light of the LSAT, is what the test tends to do. And, in the overwhelming majority of cases, the answer to the initial question is going to be D --> E. Basically, the LSAT is seeing if you can link all the premises and spot the link that's missing.

Another type of gap you might see is this:

B --> C --> D
Conclusion: A --> D

And, here you can expect A --> B to be your answer.

Finally, I can recall seeing this devil once (in a convoluted form with tons of contrapositives):

A --> B --> C
D --> E
Conclusion: A --> E

And here you can expect C --> D.

In my opinion, what you want to take away from this discussion is what you should expect, since that will get you finished quickly almost all of the time.
User avatar
 
LSAT-Chang
Thanks Received: 38
Atticus Finch
Atticus Finch
 
Posts: 479
Joined: June 03rd, 2011
 
 
trophy
Most Thankful
trophy
First Responder
 

Re: Q16 - Among Multiparty

by LSAT-Chang Thu Aug 11, 2011 7:01 pm

Thanks for the great tip, Noah! :mrgreen:
User avatar
 
bbirdwell
Thanks Received: 864
Atticus Finch
Atticus Finch
 
Posts: 803
Joined: April 16th, 2009
 
 
 

Re: Q16 - Among Multiparty

by bbirdwell Thu Aug 11, 2011 10:09 pm

Noah, I like! The Apple harvest example was fun...
I host free online workshop/Q&A sessions called Zen and the Art of LSAT. You can find upcoming dates here: http://www.manhattanlsat.com/zen-and-the-art.cfm
 
kyuya
Thanks Received: 25
Elle Woods
Elle Woods
 
Posts: 77
Joined: May 21st, 2015
 
 
 

Re: Q16 - Among multiparty democracies, those with

by kyuya Fri Jun 12, 2015 9:34 pm

I'll give this one a go.

This stimulus reminds me of a sufficient assumption in many ways. I also feel as though to successfully answer this question, it requires that we address the gap in the argument and make the connection known, which is where (C) ultimately does.

This stimulus begins with a conclusion. So we can set up the argument as follows:

Fewer # parties in democracy -> more issues each party must take a stand on --> must prioritize the issues --> and promotes tendency to compromise.

Therefore..

Those with the fewest parties will have the most productive legislatures.

The premises suggest that promoting a tendency to compromise is closely related to productivity. We can draw this by the fact the premises are supposed to support the conclusion. A degree of difficulty is added in this question because we have to realize that the conclusion is given to us in the first sentence, and the rest is simply to support it.

(A) This is unsupported, and additionally is not the assumption we are looking for.

(B) We don't know anything about number of parties being correlated with importance of compromise. Furthermore, it does not address the connection between the conclusion and premise

(C) This is where the argument leads. We begin with the conclusion, and the final premise being that about productivity. If the tendency to compromise did not make the process more productive, the argument would fall apart since that is the point it is supporting.

(D) We don't know anything about non democracies comparative to democracies, so on that basis alone we can eliminate this.

(E) NEVER is a strong word, we cannot use it here, and it also is not a required assumption.
 
josh.randall52
Thanks Received: 0
Vinny Gambini
Vinny Gambini
 
Posts: 12
Joined: December 15th, 2015
 
 
 

Re: Q16 - Among multiparty democracies, those with

by josh.randall52 Sat Jan 02, 2016 2:30 pm

M********* :|


Excuse my french. I just BR'd this PT and got to this question. I chose A because I did not correctly identify the conclusion fully. This post is more of me ranting to help myself and others. I somewhat zoned out of the most productive part as I continued reading. I was just reading about the less parties > more compromise, therefore I was honing in on an answer about more parties > more difficult to compromise. That was why I chose A.


Always find the conclusion and relate everything back to that. C is perfectly in line w/ the stimulus, and this question is quite easy.
User avatar
 
tommywallach
Thanks Received: 468
Atticus Finch
Atticus Finch
 
Posts: 1041
Joined: August 11th, 2009
 
 
 

Re: Q16 - Among multiparty democracies, those with

by tommywallach Mon Jan 04, 2016 7:08 pm

Sorry, Josh! At least it makes sense now. :)

(And apologies I had to bleep you out!).

-t
Tommy Wallach
Manhattan LSAT Instructor
twallach@manhattanprep.com
Image
 
JeremyK460
Thanks Received: 0
Elle Woods
Elle Woods
 
Posts: 80
Joined: May 29th, 2020
 
 
 

Re: Q16 - Among Multiparty

by JeremyK460 Sun Jun 21, 2020 5:50 am

noah Wrote:
Any assumption connecting A, B or C to E would technically work. But, remember that the LSAT is presenting an argument, so the idea is to link the premise to the conclusion, and so it's much more representative of what the LSAT is testing to require you use all your supporting premises in this sort of problem. Also, be aware that there could be a slight variation in the premise chain: (A --> B) --> C --> D, concluding A --> D and the connection is totally different.




I might be interpreting this wrong (and I’d love to have a conversation with someone about this!) but I think using the middle events of the argument’s logical sequence to establish the unfamiliar term in the conclusion isn’t necessary to sustain the argument’s logic. I think a premise that states something like ‘more prioritization makes the legislature more productive’ would strengthen the argument, but I feel like the argument doesn’t depend on this proposition to sustain its logic. My diagram, adding the negated premise connecting ‘prioritization’ and ‘productivity’…

P: A → B → C → D
P: B → not-E
C: A → E

It looks like the logic stops after prioritization, and that ‘fewer parties’ can’t get to ‘most productive’. But what if I proposed this…

P: A → B → C → D
P: B → not-E
P: D → not-(B → not-E)
C: A → E

I feel like this proposition (more likely to compromise makes the legislature more productive beyond any productivity that was lost by prioritization). Basically D functions as a counterbalance to B’s function, which I believe would sustain the argument’s logic, keeping the argument alive!

I’d love to talk about this with someone!